Ask Your Question
3

Finding a value so that it is perpendicular to a vector

asked 2014-07-01 13:09:26 -0600

asmith14 gravatar image

updated 2014-07-01 16:47:43 -0600

Justin gravatar image

I thought that if you are looking for a value that is perpendicular to a vector you could use the dot product and just set it equal to zero. Number 7 on the review for Exam 1 says to find a value of t so that $ \langle 1,t, t^2 \rangle$ is perpendicular to $\langle 1,1,1 \rangle $.

When I did the dot product and set it equal to zero I used the quadratic formula and it will not work because there is a negative under the radical.

$$ \langle 1,t, t^2 \rangle \cdot \langle 1, 1, 1 \rangle = 0 $$ $$ 1+t+t^2 = 0 $$

Then I put those numbers into the quadratic formula: $$ t = \frac{-1\pm\sqrt{1-4(1)(1)}}{(2)(1)} = \frac{-1\pm\sqrt{-3}}{2} $$

So am I doing it wrong, or am I doing it right but my numbers are wrong, or what is going on because this answer does not make sense to me?

edit retag flag offensive close delete

2 Answers

Sort by ยป oldest newest most voted
2

answered 2014-07-01 19:23:02 -0600

Justin gravatar image

updated 2014-07-01 21:23:42 -0600

Christina: Normally I would leave this as a comment, but perhaps this means that there are no vectors that satisfy our requirements? In other words, maybe there is no solution to this.

Comment: I tend to agree.

edit flag offensive delete publish link more
2

answered 2014-07-01 18:57:19 -0600

updated 2014-07-01 19:48:22 -0600

I don't exactly consider this an answer, but just wanted to share some thoughts and pose some more questions.

I approached this problem exactly as asmith14 did, setting the dot product of the two equal to zero. I also came up with the same answers as him/her:

$$t = \frac{-1\pm\sqrt{-3}}{2}$$

Couldn't this also be written as:

$$t = \frac{-1\pm i \sqrt{3}}{2}$$

Does this mean that there are 2 vectors that are perpendicular to $\langle1,1,1\rangle$?

Does this also mean that the perpendicular vectors would be in the complex plane? I don't remember enough about complex numbers to know if this is right...

Comment Justin, I tend to agree with you, a vector in the complex plain is one thing to consider, but in this case it makes a value of t a complex number...and correct me if I am wrong but isn't the complex plane just a way to represent the complex numbers? I don't know, it is a lot to consider...Thanks askbot!

edit flag offensive delete publish link more
Login/Signup to Answer

Question tools

Follow
1 follower

Stats

Asked: 2014-07-01 13:09:26 -0600

Seen: 37 times

Last updated: Jul 01 '14